Re: dubbio su equazione differenziale

Messaggioda manu91 » 31/12/2013, 14:57

forse mi saro espresso male ma pensavo di averla gia scritta (o per lo più non cambiava molto per me...)
$y(t)={(e^(t^2), t in t^2y<=4),(4t+1, t in t^2y>4):}$
manu91
Average Member
Average Member
 
Messaggio: 155 di 632
Iscritto il: 19/09/2011, 08:59

Re: dubbio su equazione differenziale

Messaggioda dissonance » 31/12/2013, 15:18

Ci stiamo avvicinando :-)

Ma ancora non va bene. Non ha senso scrivere $t\in t^2y\le 4$. Quella $y$ deve sparire.
dissonance
Moderatore
Moderatore
 
Messaggio: 10730 di 27760
Iscritto il: 24/05/2008, 19:39
Località: Nomade

Re: dubbio su equazione differenziale

Messaggioda manu91 » 31/12/2013, 15:47

e come?
manu91
Average Member
Average Member
 
Messaggio: 156 di 632
Iscritto il: 19/09/2011, 08:59

Re: dubbio su equazione differenziale

Messaggioda dissonance » 31/12/2013, 16:02

E' esattamente questo il punto che cerco di sottolineare dall'inizio del thread. Questo esercizio non è meccanico, perché per capire il "come" occorre fermarsi a pensare un po'. Provaci. Che cos'è una soluzione di una equazione differenziale? Riflettendo su questo capirai "come" si fa a costruire la soluzione che ti serve.
dissonance
Moderatore
Moderatore
 
Messaggio: 10731 di 27760
Iscritto il: 24/05/2008, 19:39
Località: Nomade

Re: dubbio su equazione differenziale

Messaggioda gugo82 » 01/01/2014, 22:25

D'altra parte, un problema simile l'avevamo risolto qui, esaminando tutte le problematiche connesse con la presenza di un secondo membro della EDO definito per casi.
Nel caso già esaminato, il problema era più semplice, perché il secondo membro dipendeva solo da \(t\); qui, invece, hai una dipendenza da \(y\) di cui devi "disfarti" in qualche modo, se vuoi scrivere la soluzione del PdC.

Per capire come fare, potrebbe essere utile (ma è solo un'idea, non ho ancora esplorato la questione) fare uno studio qualitativo della soluzione prima di tentare di determinare la soluzione stessa.
Uno studio qualitativo, ad esempio, orientato a stabilire se, per caso, l'integrale del PdC assegnato soddisfa sempre (o almeno in un intorno di \(0\) "sensatamente grande") una relazione del tipo \(t^2 y(t)\leq 4\).
Sono sempre stato, e mi ritengo ancora un dilettante. Cioè una persona che si diletta, che cerca sempre di provare piacere e di regalare il piacere agli altri, che scopre ogni volta quello che fa come se fosse la prima volta. (Freak Antoni)
Avatar utente
gugo82
Cannot live without
Cannot live without
 
Messaggio: 15621 di 44972
Iscritto il: 12/10/2007, 23:58
Località: Napoli

Re: dubbio su equazione differenziale

Messaggioda manu91 » 02/01/2014, 11:08

ciao @gugo82 :)
si si mi ricordo di quell'esercizio:)
piu che altro mi era venuto un dubbio, se dovendo valutare la funzione in un punto potevo tranquillamente "sostituire" il valore della t oppure dovevo stare attento a qualcosa:)
manu91
Average Member
Average Member
 
Messaggio: 159 di 632
Iscritto il: 19/09/2011, 08:59

Re: dubbio su equazione differenziale

Messaggioda gugo82 » 05/01/2014, 16:32

No, non puoi "tranquillamente sostituire"... Prima devi "tranquillamente ragionare" su cosa succede, poi sostituisci. :wink:

Un piccolo studio qualitativo:
Testo nascosto, fai click qui per vederlo
Il PdC assegnato è:
\[
\tag{1}
\left\{ \begin{split}
y^\prime (t) &= 3\ \min \left\{ t^2\ y(t), 4\right\} \\
y(0) &= 1\; .
\end{split}\right.
\]
La funzione a secondo membro, i.e.:
\[
f(t,y) := 3\ \min \left\{ t^2\ y, 4\right\} = \begin{cases} 12 &\text{, se } 3\ t^2y\geq 4\\ t^2y &\text{, se } t^2y\leq 4\; ,\end{cases}
\]
è definita e continua in \(\mathbb{R}^2\), nonché localmente lipschitziana rispetto ad \(y\); quindi (1) ha un'unica soluzione locale che si prolunga in un unica soluzione massimale.
Dato che \(f\) è di classe \(C^\infty(\mathbb{R}^2\setminus \Gamma)\), in cui:
\[
\Gamma:= \left\{ (t,y)\in \mathbb{R}^2:\ t^2y=4 \right\} \; ,
\]
ma che \(f\) non ha maggiore regolarità di \(C^0\) in tutto \(\mathbb{R}^2\)1, la soluzione massimale di (1) è solo una funzione di classe \(C^1\) nel proprio intervallo di definizione.
Tuttavia, negli intervalli in cui il grafico della soluzione non interseca punti della curva \(\Gamma\), la soluzione è \(C^\infty\).
Il secondo membro della EDO è sublineare rispetto ad \(y\), poiché infatti:
\[
|f(t,y)|\leq 3 t^2\ |y|\; ,
\]
e ciò assicura che le soluzioni massimali della EDO sono definite su tutto \(\mathbb{R}\).

La EDO ha inoltre una soluzione stazionaria globale, cioé \(y^*(t)=0\). Dato che siamo in regime di unicità, il grafico di ogni altra soluzione massimale della EDO non può incontrare il grafico di \(y^*\); conseguentemente, ogni altra soluzione massimale della EDO o assume sempre valori positivi, oppure assume sempre valori negativi in \(\mathbb{R}\).

Inoltre, si ha \(f(t,y)> 0\) solo se \((t,y)\) è in:
\[
\begin{split}
\Omega^+ &:= \{(t,y)\in \mathbb{R}^2:\ t^2 y\geq 4\} \cup \{(t,y)\in \mathbb{R}^2:\ t^2y\leq 4 \text{ e } t^2y> 0\}\\
&=\{(t,y)\in \mathbb{R}^2:\ t^2 y\geq 4\} \cup \{(t,y)\in \mathbb{R}^2:\ t^2y\leq 4,\ t\neq 0 \text{ e } y> 0\}\\
&= (\mathbb{R}\setminus \{0\})\times ]0,\infty[
\end{split}
\]
(sicché \(\Omega^+\) è l'unione del primo e del secondo quadrante) ed analogamente \(f(t,y)< 0\) solo se \((t,y)\) è in:
\[
\begin{split}
\Omega^- &:= \{(t,y)\in \mathbb{R}^2:\ t^2y\leq 4 \text{ e } t^2y<0\}\\
&= \{(t,y)\in \mathbb{R}^2:\ t^2y\leq 4,\ t\neq 0 \text{ e } y<0\}\\
&= (\mathbb{R}\setminus \{0\})\times ]-\infty ,0[
\end{split}
\]
(ossia \(\Omega^-\) è l'unione del terzo e del quarto quadrante), mentre \(f(t,y)=0\) solo se \(t=0\) oppure \(y=0\) cioé se \((t,y)\in \Omega_0:= \mathbb{R}^2\setminus (\Omega^+ \cup \Omega^-)\) (l'insieme \(\Omega_0\) è l'unione degli assi coordinati).
Ne consegue che la soluzione massimale è strettamente crescente [risp. decrescente] fintantoché il suo grafico cade nella zona \(\Omega^+\cup \Omega_0\) [risp. \(\Omega^-\cup \Omega_0\)].
        Internet Explorer richiede Adobe SVG Viewer per visualizzare il grafico



In più, finché il grafico della soluzione massimale è in \(\mathbb{R}^2\setminus \Gamma\) si ha:
\[
\begin{split}
y^{\prime \prime} (t) &= \begin{cases} 0 &\text{, se } t^2y(t)>4\\
6ty(t) + 3t^2y^\prime (t) &\text{, se } t^2y(t)<4
\end{cases}\\
&= \begin{cases} 0 &\text{, se } t^2y(t)>4\\
6ty(t) + 3t^2\min \{t^2 y(t),4\} &\text{, se } t^2y(t)<4
\end{cases}\\
&= \begin{cases} 0 &\text{, se } t^2y(t)>4\\
6ty(t) + 3t^4y(t) &\text{, se } t^2y(t)<4
\end{cases}\\
&= \begin{cases} 0 &\text{, se } t^2y(t)>4\\
3ty(t)\ (2+t^3) &\text{, se } t^2y(t)<4\; .
\end{cases}
\end{split}
\]
Da ciò segue che la generica soluzione massimale è un polinomio di primo grado fintantoché il suo grafico rimane confinato nella regione descritta dalla limitazione \(t^2y\geq 4\); d'altra parte, fino a che il grafico rimane confinato nella regione descritta dalla limitazione \(t^2 y\leq 4\), la soluzione massimale può cambiare concavità. La situazione è descritta nel diagramma che segue:
        Internet Explorer richiede Adobe SVG Viewer per visualizzare il grafico


in cui \(\cup\) sta per "convesso" e \(\cap\) per "concavo".

Queste sono solo alcune tra le informazioni qualitative che possiamo dire circa gli integrali della EDO assegnata nel PdC; ora proviamo a risolvere esplicitamente il problema.
Testo nascosto, fai click qui per vederlo
Visto che il punto iniziale \((t_0,y_0)=(0,1)\) è in \(\Omega_0\) e che la soluzione locale \(y_1(t)\) di (1), definita in un intorno completo di \(0\), ha il grafico che localmente si discosta poco da tale punto, possiamo senz'altro supporre che il grafico di \(y_1(t)\) cada tutto nella zona \(\Omega^+\cup \Omega_0\): pertanto \(y_1(t)\) è crescente intorno a \(0\); d'altro canto, a meno di restringere un po' l'intervallo di definizione, possiamo supporre che \(y_1(t)\) sia concava a sinistra e convessa a destra di \(0\).
Inoltre, dato che il punto iniziale \((0,1)\) cade nella zona in cui \(t^2 y\leq 4\), a meno di restringere ancora un po' l'intervallo di definizione, possiamo supporre che risulti \(t^2 y_1(t)\leq 4\) sempre intorno a \(0\); ciò porta a concludere che \(f(t,y_1(t))=3t^2\ y_1(t)\) cosicché \(y_1(t)\) soddisfa:
\[
y_1^\prime (t) = 3t^2\ y_1(t)\; ;
\]
da ciò, separando le variabili2 e tenendo presente la condizione iniziale, si trae:
\[
\int_1^{y_1(t)} \frac{1}{\eta}\ \text{d} \eta = \int_0^t 3\tau^2\ \text{d} \tau
\]
ossia:
\[
\ln y_1(t) = t^3
\]
da cui:
\[
y_1(t)=e^{t^3}
\]
per \(t\) in un conveniente intorno di \(0\).

Ora che abbiamo l'espressione analitica dell'integrale locale, bisogna crecare di prolungarlo ad un integrale massimale.
Per fare ciò, bisogna innanzitutto capire "fin dove può valere" l'espressione analitica trovata, cioé determinare il conveniente intorno di \(0\) citato sopra.
Dalla EDO segue che l'espressione analitica trovata vale fintantoché non cambia il secondo membro dell'equazione; dato che il secondo membro della EDO cambia non appena \(t^2y_1(t)\) diventa \(\geq 4\), è abbastanza naturale concludere che il conveniente intorno di \(0\) citato sopra è il più grande intorno di \(0\) che si può isolare nell'insieme delle soluzioni della disequazione:
\[
\tag{2}
t^2\ e^{t^3}\leq 4\; ;
\]
pertanto abbiamo bisogno di determinare (più o meno esplicitamente) l'insieme delle soluzioni di tale disequazione.
La funzione \(\phi (t):=t^2e^{t^3}\) è nonnegativa, divergente in \(+\infty\) ed il suo diagramma come asintoto orizzontale a sinistra l'asse delle \(t\); d'altra parte, essa è crescente [risp. decrescente] in \(]-\infty, -\sqrt[3]{2/3}]\cup [0,\infty[\) [risp. \([-\sqrt[3]{2/3},0]\)] ed il suo massimo locale, preso in \(-\sqrt[3]{2/3}\), è \(\phi (-\sqrt[3]{2/3}) \approx 0.39\).
Ne viene che la disequazione (2) è soddisfatta certamente per \(t\leq T\), in cui \(T>0\) è l'unico numero reale tale che \(T^2e^{T^3}=4\); tale numero, non esprimibile elementarmente, ha il valore \(T\approx 1.075\).
Conseguentemente, l'espressione locale della soluzione \(y_1(t)=e^{t^3}\) vale in tutta la semiretta \(]-\infty, T]\).
[E già questo basterebbe per risolvere l'esercizio, perché \(-1\in ]-\infty,T]\).]

Subito dopo \(T\), la EDO soddisfatta dall'integrale massimale in cui si prolunga la soluzione locale del PdC assegnato è:
\[
y^\prime (t) = 12\; ,
\]
ed ad essa va accoppiata la condizione iniziale:
\[
y(T)=\frac{4}{T^2}\; .
\]
La soluzione del nuovo PdC:
\[
\begin{cases}
y^\prime (t) = 12 \\
y(T) = 4/T^2
\end{cases}
\]
è evidentemente:
\[
y_2(t) = 12t + \frac{4}{T^2} \left( 1- 3 T^3 \right)
\]
e si vede senza sforzo che essa è in forza per tutti gli \(t\geq T\).

Da quanto detto si conclude che la soluzione massimale del PdC assegnato è la funzione che si ottiene raccordando \(y_1\) ed \(y_2\), cioé:
\[
y(t;0,1) := \begin{cases} e^{t^3} &\text{, se } t\leq T\\
12t + \frac{4}{T^2} \left( 1- 3 T^3 \right) &\text{, se } t\geq T\; .
\end{cases}
\]
        Internet Explorer richiede Adobe SVG Viewer per visualizzare il grafico


Note

  1. Infatti, è semplice verificare che le derivate parziali di \(f\) non esistono sulla curva \(\Gamma\), che costituisce la frontiera dell'insieme di regolarità di \(f\).
  2. Cosa lecita, perché essendo \(y_0=1> 0\), per quanto detto all'inizio la soluzione \(y(t)\) è sempre positiva.
Sono sempre stato, e mi ritengo ancora un dilettante. Cioè una persona che si diletta, che cerca sempre di provare piacere e di regalare il piacere agli altri, che scopre ogni volta quello che fa come se fosse la prima volta. (Freak Antoni)
Avatar utente
gugo82
Cannot live without
Cannot live without
 
Messaggio: 15645 di 44972
Iscritto il: 12/10/2007, 23:58
Località: Napoli

Precedente

Torna a Analisi matematica di base

Chi c’è in linea

Visitano il forum: Nessuno e 1 ospite